Proving the Even Degree Property of Vertices in Closed Trails

Click For Summary
All vertices in a closed trail must have even degree due to the nature of how trails connect vertices. In a closed trail, each vertex is entered and exited, resulting in an even count of edges incident to that vertex. Conversely, non-closed trails have endpoints that lead to vertices with odd degrees, as they only connect to one edge. The discussion suggests that deleting a vertex from a closed trail shortens it without altering the even degree property of the remaining vertices. A coherent proof can be constructed by demonstrating that the entry and exit of edges at each vertex maintains even degrees throughout the trail.
tarheelborn
Messages
121
Reaction score
0

Homework Statement



All vertices in a closed trail have even degree.

Homework Equations


The Attempt at a Solution



Intuitively, I know this statement is true, but I can't seem to see a clear way to show it. I know that a closed trail is a path that connects vertices, so one would follow an edge through a vertex to another edge, thus indicating that, on this particular path, this particular vertex has degree two. Clearly, a non-closed trail has endpoints which must have odd degree because they reach a stopping point on the trail, with a vertex incident to only one edge. Will you please help me put these facts into a coherent proof? Thank you.
 
Physics news on Phys.org
Hint: how could you make a closed trail shorter? Also, be sure you know exactly what a closed trail is.
 
We could make a closed trail shorter by deleting a vertex. If we delete a vertex, the edges incident to that vertex are also deleted, so the trail would be shorter. But this action wouldn't necessarily affect the degree of vertices, would it?
 
Question: A clock's minute hand has length 4 and its hour hand has length 3. What is the distance between the tips at the moment when it is increasing most rapidly?(Putnam Exam Question) Answer: Making assumption that both the hands moves at constant angular velocities, the answer is ## \sqrt{7} .## But don't you think this assumption is somewhat doubtful and wrong?

Similar threads

Replies
11
Views
5K
Replies
1
Views
2K
  • · Replies 2 ·
Replies
2
Views
5K
Replies
3
Views
3K
  • · Replies 5 ·
Replies
5
Views
3K
  • · Replies 5 ·
Replies
5
Views
14K
  • · Replies 2 ·
Replies
2
Views
2K
  • · Replies 3 ·
Replies
3
Views
3K
Replies
3
Views
2K
  • · Replies 1 ·
Replies
1
Views
4K